Sei sulla pagina 1di 121

GENERAL LABORATORY RULES AND PRECAUTIONS

FOR ELECTRICAL SAFETY

The following general rules and precautions are to be observed at all times in the
laboratory. These rules are for the benefit of the experimenter as well as those around
him/her. Additional rules and precautions may apply to a particular laboratory.

1. There must be at least two (2) people in the laboratory while working onlive circuits or
chemical processing.

2. Shoes must be worn at all times.

3. Remove all loose conductive jewelry and trinkets, including rings, which may come in
contact with exposed circuits. (Do not wear long loose ties, scarves, or other loose clothing
around machines.)

4. Consider all circuits to be "hot" unless proven otherwise.

5. When making measurements, form the habit of using only one hand at a time. No part of
a live circuit should be touched by the bare hand.

6. Keep the body, or any part of it, out of the circuit. Where interconnecting wires and
cables are involved, they should be arranged so people will not trip over them.

7. Be as neat a possible. Keep the work area and workbench clear of items not used in the
experiment.

8. Always check to see that the power switch is OFF before plugging into the outlet. Also,
turn instrument or equipment OFF before unplugging from the outlet.

9. When unplugging a power cord, pull on the plug, not on the cable.

10. When disassembling a circuit, first remove the source of power.

11. "Cheater" cords and 3-to-2 prong adapters are prohibited unless an adequate separate
ground lead is provided, the equipment or device is double insulated, or the laboratory
ground return is known to be floating.

12. No ungrounded electrical or electronic apparatus is to be used in the laboratory unless


it is double insulated or battery operated.

13. Keep fluids, chemicals, and beat away from instruments and circuits.

14. Report any damages to equipment, hazards, and potential hazards to the laboratory
instructor.
15. If in doubt about electrical safety, see the laboratory instructor. Regarding specific
equipment, consult the instruction manual provided by the manufacturer of the equipment.
Information regarding safe use and possible- hazards should be studied carefully.

Electrical Safety in the Lab

While electricity is in constant use by the researcher, both within and outside the laboratory,
significant physical harm or death may result from its misuse.

With direct current, a person can detect a "tingling" feeling at 1 mA and the median "let-go"
threshold (the current at which one cannot release the conductor) is 76 mA. For 60 Hertz
alternating current, the values are 0.4 mA and 16 mA, respectively. Women are more sensitive to
the effects of electrical current; approximately 2/3 of the current is needed to produce the same
effect. Higher currents produce respiratory inhibition, then ventricular fibrillation, and ultimately
cardiac arrest.

If an electrical hazard is suspected, the device in question should be disconnected immediately


and the cause ascertained by a person competent in such matters. Work on electrical devices
should be done only after the power has been shut off in such a manner that it cannot be turned
on accidentally. Since malfunctioning equipment may contain shorts, merely turning off the
equipment is not sufficient to prevent accidents. Equipment should be unplugged before being
inspected or the circuit the equipment is wired to deactivated by putting the circuit breaker in the
off position or removing the fuse. Equipment wired to a safety switch should be turned off at the
safety switch. Internal energy storage devices such as capacitors must be discharged.

All electrical wiring and construction must conform to standard safety practice. The
minimum safety practice must conform to the coded regulations of the City of Columbus and the
State of Ohio. High voltage equipment must be labeled: Danger, High Voltage. Switches to turn
off all electrical power to the equipment in case of emergency should be prominently labeled.

The following are a list of rules for working with electrical equipment:

1. Turn off the power to equipment before inspecting it. Turn off circuit breakers or unplug the
equipment. To turn off a safety switch, use your left hand (wear insulating gloves made of
leather or heavy cotton or rubber), turn your face away from the box, and pull the handle
down. Circuits may discharge violently when being turned on or off and the cover to the
junction box may be blown open.

2. Use only tools and equipment with non-conducting handles when working with electrical
devices.

3. All current transmitting parts of any electrical devices must be enclosed.


4. When checking an operating circuit, keep one hand either in a pocket or behind your back to
avoid making a closed circuit through the body.

5. Maintain a work space clear of extraneous material such as books, papers, and clothes.

6. Never change wiring with circuit plugged into power source.

7. Never plug leads into power source unless they are connected to an established circuit.

8. Avoid contacting circuits with wet hands or wet materials.

9. Wet cells should be placed on a piece of non-conducting material.

10. Check circuits for proper grounding with respect to the power source.

11. Do not insert another fuse of larger capacity if an instrument keeps blowing fuses - this is a
symptom requiring expert repairs. If a fuse blows, find the cause of the problem before
putting in another one.

12. By the Ohio Fire Code, extension cords must be connected to a power strip equipped with a
fuse.

13. Do not use or store highly flammable solvents near electrical equipment.

14. Multi-strip outlets (cube taps) should not be used in place of permanently installed
receptacles. If additional outlets are required have them installed by an electrician.

15. Keep access to electrical panels and disconnect switches clear and unobstructed.

Static Electricity and Spark Hazards:

Sparks may result in explosions in areas where flammable liquids are being used and therefore
proper grounding of equipment and containers is necessary. Some common potential sources of
sparks are:

1. The making and braking of an electrical circuit when the circuit is energized.

2. Metal tanks and containers.

3. Plastic lab aprons.


4. Metal clamps, nipples, or wire used with nonconducting hoses.

5. High pressure gas cylinders upon discharge.

ELECTRICAL PRACTICE WORKSHOP / LABORATORY


FACULTY OF ELECTRICAL ENGINEERING
UNIVERSITY TEKNOLOGI MALAYSIA
INTRODUCTION AND SAFETY PRECAUTIONS
Electrical danger cannot be assumed easy. The danger is difficult to trace since the flow
of current cannot be seen. If the current flow in the wrong direction, the current can
defect human body, cause a shock, paralyzed, fire, explosion, death and others. This
accident can be prevented by observing / obeying safety procedure / rule. This safety rule
is intended to protect employee, user, equipment and building from danger and risk due
to electrical effect. These rules are base on IEE (Institution of Electrical Engineers)
regulation and Department of Electric and Gas Supply that have been updated from to
time.
The followings are safety procedures/ rules which must be obey:
a) The floor of workplace must be free (clean) from oil, water and grease. These
materials can cause the worker to slip while working there.
b) Equipment used must in good and perfect condition. If not, report to officer in
charge. All equipment must be kept at right and safe place so that it is easy to look
for especially during emergency. After use, the equipment must be kept at its
original place.
c) Wear suitable clothing, not too tight and not too loose. Wear shoe having thick
and all round sew sole when doing wiring work.
Wearing slipper is strictly prohibited.
d) Nobody is allowed to make joke or playing sharp instrument or object with friend
while doing wiring work.
e) Make sure the cable/conductor used fulfill its size (rating) and having suitable
insulation.
f) Make sure every electrical installation have effective earthing and avoid it from
rust.
g) Any addition of circuit must be avoided unless there is permission from officer in
charge.
h) Do not dismantle electric component/device used in the experiment without the
knowledge of officer in charge.
i) Assistance from officer in charge must sought before testing the experiment with
electrical supply,
j) After using electrical machine, it must be switch off.
k) In case any accident happened, officer in charge must be informed immediately.
l) All electrical supply must off after finishing the job or before leaving laboratory.
m) Before fitting plug to the socket, socket outlet switch must be in the off condition.
n) Make sure environment around workplace is clean and systematic before and
after work.

ELECTRICITY FUNDAMENTALS

Electricity is an apparent force in nature that exists whenever there is a net electrical
charge between any two objects.

Basics of Electrostatics:

Electrical charges are either negative (electrons) or positive (protons)


The unit of charge, q , is called the coloumb.
When there are equal numbers of positive and negative charges there is no electrical
force as there is no net charge. This is the case for a neutral atom.
Electrical force is created when electrons are transferred from one material to
another (e.g. rubbing a wool cloth with a plastic comb).
Electrical charge is conserved; charge is neither created nor destroyed

Properties of Electricity:

CURRENT: denoted by I and measured in amperes. Current flows


from negativelycharged material to positively charged material and is essentially the
number of electrons per second that are carried through a conductor. Current is
measured in units of amps. 1 amp = 1 coloumb/sec = 6.2 x 1018 electrons per second!

VOLTAGE: Potential difference between a negatively charged object and a


positively charged one (like two terminals on a battery). Potential difference is
measured in units of Volts ( V )which represents the work done per unit charge to
move electrons between the positive and negative terminals. If a potential difference
exists, then energy can be extracted.

Imagine that you have two opposite charges that you want to separate. It takes work
to separate the charge and thus the separated charges store energy. The amount of
stored energy is given by:

E = qV where V is the voltage or electric potential of some system.

The units of voltage or Volts: 1 Volt = 1 Joule/Coulomb

If the separated charges get back together, work/energy can be extracted from the
system. If there is some pathway for the charges to flow then we get a current.
Current is denoted by I and is in units of amperes or amps 1 Ampere = 1
coulomb/second

RESISTANCE: Property of material that helps prevent the flow of electrons in it.
Metals are good conductors due to low resistance. Wood is a poor conductor due to
high resistance. Resistance, R , is measured in ohms and depends upon both the type
of material and its size. Long wires have more resistance than short wires; thin
wires have more resistance than thick wires. R is also temperature dependent.

OHM's LAW

Is there a relation between I, V, and R ? Let's do an experiment:

In the above circuit there is a battery (V), some resistors (R), and a light bulb that can only
be activated if the right number of amps reach it. We can control this buy putting the right
resistance in the circuit for a given battery Voltage.

Experimental results then lead to Ohms law:

V=R*I

This is a linear relation. If you double the voltage (V) then for the same value of R you get
twice the current. If you want to keep the current the same value after doubling V, you
would have to double the resistance (R).
Example:

Standard US household voltage is 120 Volts.


The heating element in your toaster has R = 15 ohms.
What is the current flowing through your toaster?

I = V/R = 120/15 = 8 amps

Your electricity bill essentially measures the amount of current that you use but you use
this current as Power .

Power = V * I

So the toaster has a power of 120x8 = 960 Watts.

Energy = Power * Time (and its energy --> kilowatt hours that you pay for - a 100 watt
light bulb left on 10 hours = 1 kilowatt hour. )

If you leave your toaster on for one hour, than that would also be approximately 1 KWH
(960 watt-hours if you want to nit pick).

1000 Watt-hours = 1 Kilowatt hour (KWH); A KWH will be our basic unit of energy
in this class. You purchase KWHs from the electric utility whenever you use power in your
home.

The Discovery of Electricy and Magnetism and the Generation of Electricity.

In the early 19th century the following similarity between two charged particles and two
magnets was observed:

both created "forces" that could operate in a vacuum


charge had a postive and negative component; magnets had a north and south
pole force could then be either attractive or repulsive.
both the magnetic force and the electrostatic force strength decreased as 1/R2

In 1820 Oersted did this experiment:


and discovered that an electric current creates a magnetic field

Similarly, a coil of wire with a current passing through it generates a magnetic field. This is
known as an electromagnet or solenoid .

So now we know that a current can create a magnetic field. If a magnetic field can create a
current then we have a means of generating electricity. Experiments showed that a
magnetic just sitting next to a wire produced no current flow through that wire. However,
if the magnet is moving a current is induced in the wire. The faster the magnet moves, the
greater the induced current.

This is the principal behind simple electric generators in which a wire loop is rotated
between to stationary magnetics. This produces a continuously varying voltage which in
turn produces an alternating current .

Diagram of a simple electric generator:


In this position there is no current
flow but their is a large potential
difference (a large voltage)

In this position the Voltage is now


zero and the current flow is at a
maximum

To generate electricty then, all we really want to do is have some (mechanical) mechanism
turn a crank that rotates a loop of wire between stationary magnets. The faster we can get
this crank turned, the more current we can generate.

Popular Methods of Turning the Crank:

Let water fall on it (Hydro Power)


Direct a nozzle of steam at it (Coal or Nuclear Fired Steam Plant)
Let the wind turn it (windmill)
Why do transmission lines carry such high voltages?

Consider the following:

Electricity is generated at the generating plant at 120 Volts and then delivered to the
households over conductors.
There are 10 households and each needs 1000 Watts (for their toasters)
The electric company must therefore supply 10x1000 = 10,000 Watts.
Power = I x V I = P/V I = 10000/120 = 83 amps
But, electrical power is dissipated as heat according to P = I2R (subsitute V=RI from
ohms law in above)
Lets assume R= 1: We now have heat dissipation = (83.3)*(83.3)(1) = 6944
watts. Heat dissipation is energy lost by the system. This loss is unavoidable!

To deliver the 10,000 watts that the consumer needs requires that we generate
16,944 watts and hence have an overall efficiency of 10,000/16,944 = 59%
which the consumer would pay for

How to solve the loss problem:

Current = Power/Voltage; If we increase V by a factor of 10, then I lowers by a factor of 10


(at constant power) and the power dissipated as heat lowers by a factor of 102.

Hence if we increase 120 Volts to 1200 Volts we have only 69.4 watts of energy loss and a
99% energy efficient delivery system This is why high voltage (typically 760 thousand
Volts or 760 kiloVolts) transmission lines are required to delivery electricity from central
generating sources (e.g. a hydroelectric dam) to consumers/grids hundreds of miles away.

How to change the voltage: Use a Transformer


A transformer uses alternating current in one coil to
induce alternating current in another. The induced
voltage is given by: Vout = Vin x N2/N1 where N1 =
Number of coils in the Primary and N2= Number of
coils in the secondary. When N2 is less than N1, we
reduce Vout. This is why there are transformers on
power lines to step the voltage down to 120 Volts by
the time it reaches your house.

Energy conservation tells us that Power In = Power Out

so

Vout x Iout = Vin x Iin

Since Vin is very high, Iin is low and (to prevent transmission loss); when Vin is stepped
down to produce Vout (what you get at your house), Iout increases so you can run your stuff.

And that's the way the world works.

Electrical Formulas
Electrical is the branch of physics dealing with electricity, electronics and electromagnetism.
Electrical formulas play a great role in finding the parameter value in any electrical circuits.
Most commonly used electrical formulas are formulas related to voltage, current, power,
resistance etc.

Below are given some commonly used Electrical formulas which may be helpful for you.

Quantity Formula Unit

Charge Q=CV Coulomb (C)

Capacitance Farad (F)


C = QV
Inductance Henry (L or H)
VL = - L didt

Voltage V=IR Volt (V)

Current Ampere (A)


I = VR

Resistance R = VI ohm ()

Power P = VI Watt (W)

Conductance G = 1R mho ()

Impedance Z2 = R2 + (xL - xc)2 ohm ()

Resonant Frequency f = 12LC Hertz (Hz)

Electrical Formulas helps us to calculate the parameters related to electricals in any electrical
components.

Table of Electrical Symbols


Symbol Component name Meaning

Wire Symbols

Electrical Wire Conductor of electrical current

Connected Wires Connected crossing

Not Connected Wires Wires are not connected

Switch Symbols and Relay Symbols

SPST Toggle Switch Disconnects current when open


SPDT Toggle Switch Selects between two connections

Pushbutton Switch (N.O) Momentary switch - normally open

Pushbutton Switch (N.C) Momentary switch - normally closed

DIP Switch DIP switch is used for onboard configuration

SPST Relay

Relay open / close connection by an electromagnet

SPDT Relay

Jumper Close connection by jumper insertion on pins.

Solder Bridge Solder to close connection

Ground Symbols

Used for zero potential reference and electrical shock


Earth Ground
protection.

Chassis Ground Connected to the chassis of the circuit

Digital / Common Ground


Resistor Symbols

Resistor (IEEE)

Resistor reduces the current flow.

Resistor (IEC)

Potentiometer (IEEE)

Adjustable resistor - has 3 terminals.

Potentiometer (IEC)

Variable Resistor /
Rheostat (IEEE)
Adjustable resistor - has 2 terminals.
Variable Resistor /
Rheostat (IEC)

Trimmer Resistor Preset resistor

Thermal resistor - change resistance when


Thermistor
temperature changes

Photoresistor / Light dependent Photo-resistor - change resistance with light intensity


resistor (LDR) change

Capacitor Symbols

Capacitor is used to store electric charge. It acts as


Capacitor
short circuit with AC and open circuit with DC.
Capacitor

Polarized Capacitor Electrolytic capacitor

Polarized Capacitor Electrolytic capacitor

Variable Capacitor Adjustable capacitance

Inductor / Coil Symbols

Inductor Coil / solenoid that generates magnetic field

Iron Core Inductor Includes iron

Variable Inductor

Power Supply Symbols

Voltage Source Generates constant voltage

Current Source Generates constant current.

AC Voltage Source AC voltage source


Electrical voltage is generated by mechanical rotation
Generator
of the generator

Battery Cell Generates constant voltage

Battery Generates constant voltage

Generates voltage as a function of voltage or current


Controlled Voltage Source
of other circuit element.

Generates current as a function of voltage or current


Controlled Current Source
of other circuit element.

Meter Symbols

Measures voltage. Has very high resistance.


Voltmeter
Connected in parallel.

Measures electric current. Has near zero resistance.


Ammeter
Connected serially.

Ohmmeter Measures resistance

Wattmeter Measures electric power

Lamp / Light Bulb Symbols

Lamp / light bulb Generates light when current flows through


Lamp / light bulb

Lamp / light bulb

Diode / LED Symbols

Diode allows current flow in one direction only (left


Diode
to right).

Allows current flow in one direction, but also can


Zener Diode flow in the reverse direction when above breakdown
voltage

Schottky Diode Schottky diode is a diode with low voltage drop

Varactor / Varicap Diode Variable capacitance diode

Tunnel Diode

Light Emitting Diode (LED) LED emits light when current flows through

Photodiode Photodiode allows current flow when exposed to light

Transistor Symbols

Allows current flow when high potential at base


NPN Bipolar Transistor
(middle)
Allows current flow when low potential at base
PNP Bipolar Transistor
(middle)

Made from 2 bipolar transistors. Has total gain of the


Darlington Transistor
product of each gain.

JFET-N Transistor N-channel field effect transistor

JFET-P Transistor P-channel field effect transistor

NMOS Transistor N-channel MOSFET transistor

PMOS Transistor P-channel MOSFET transistor

Misc. Symbols

Motor Electric motor

Transformer Change AC voltage from high to low or low to high.

Electric bell Rings when activated

Buzzer Produce buzzing sound

The fuse disconnects when current above threshold.


Fuse
Used to protect circuit from high currents.
Fuse

Bus

Bus Contains several wires. Usually for data / address.

Bus

Optocoupler / Opto-isolator Optocoupler isolates connection to other board

Loudspeaker Converts electrical signal to sound waves

Microphone Converts sound waves to electrical signal

Operational Amplifier Amplify input signal

Schmitt Trigger Operates with hysteresis to reduce noise.

Analog-to-digital converter
Converts analog signal to digital numbers
(ADC)

Digital-to-Analog converter
Converts digital numbers to analog signal
(DAC)
Crystal Oscillator Used to generate precise frequency clock signal

Antenna Symbols

Antenna / aerial

Transmits & receives radio waves

Antenna / aerial

Dipole Antenna Two wires simple antenna

Logic Gates Symbols

NOT Gate (Inverter) Outputs 1 when input is 0

AND Gate Outputs 1 when both inputs are 1.

NAND Gate Outputs 0 when both inputs are 1. (NOT + AND)

OR Gate Outputs 1 when any input is 1.

NOR Gate Outputs 0 when any input is 1. (NOT + OR)

XOR Gate Outputs 1 when inputs are different. (Exclusive OR)


D Flip-Flop Stores one bit of data

Multiplexer / Mux 2 to 1

Connects the output to selected input line.

Multiplexer / Mux 4 to 1

Demultiplexer / Demux 1 to 4 Connects selected output to the input line.

Wires and connections


Component Circuit Symbol Function of Component

To pass current very easily from one part of a


Wire
circuit to another.

A 'blob' should be drawn where wires are


connected (joined), but it is sometimes omitted.
Wires joined Wires connected at 'crossroads' should be
staggered slightly to form two T-junctions, as
shown on the right.

In complex diagrams it is often necessary to


draw wires crossing even though they are not
connected. The simple crossing on the left is
Wires not joined
correct but may be misread as a join where the
'blob' has been forgotten. The bridge symbol on
the right leaves no doubt!

Power Supplies
Component Circuit Symbol Function of Component
Supplies electrical energy.
The larger terminal (on the left) is positive (+).
Cell
A single cell is often called a battery, but strictly a
battery is two or more cells joined together.

Supplies electrical energy. A battery is more than


Battery one cell.
The larger terminal (on the left) is positive (+).

Converts light to electrical energy.


Solar Cell
The larger terminal (on the left) is positive (+).

Supplies electrical energy.


DC supply DC = Direct Current, always flowing in one
direction.

Supplies electrical energy.


AC supply AC = Alternating Current, continually changing
direction.

A safety device which will 'blow' (melt) if the


Fuse current flowing through it exceeds a specified
value.

Two coils of wire linked by an iron core.


Transformers are used to step up (increase) and step
Transformer down (decrease) AC voltages. Energy is transferred
between the coils by the magnetic field in the core.
There is no electrical connection between the coils.

A connection to earth. For many electronic circuits


Earth this is the 0V (zero volts) of the power supply, but
(Ground) for mains electricity and some radio circuits it
really means the earth. It is also known as ground.

Output Devices: Lamps, Heater, Motor, etc.


Component Circuit Symbol Function of Component
A transducer which converts electrical energy to
light. This symbol is used for a lamp providing
Lamp (lighting)
illumination, for example a car headlamp or
torch bulb.

A transducer which converts electrical energy to


light. This symbol is used for a lamp which is an
Lamp (indicator)
indicator, for example a warning light on a car
dashboard.

A transducer which converts electrical energy to


Heater
heat.

A transducer which converts electrical energy to


Motor
kinetic energy (motion).

A transducer which converts electrical energy to


Bell
sound.

A transducer which converts electrical energy to


Buzzer
sound.

A coil of wire which creates a magnetic field


when current passes through it. It may have an
Inductor
iron core inside the coil. It can be used as a
(Coil, Solenoid)
transducer converting electrical energy to
mechanical energy by pulling on something.

Switches
Component Circuit Symbol Function of Component

Push Switch A push switch allows current to flow only


(push-to- when the button is pressed. This is the switch
make) used to operate a doorbell.
This type of push switch is normally closed
Push-to-Break
(on), it is open (off) only when the button is
Switch
pressed.

SPST = Single Pole, Single Throw.


On-Off Switch
An on-off switch allows current to flow only
(SPST)
when it is in the closed (on) position.

SPDT = Single Pole, Double Throw.


A 2-way changeover switch directs the flow of
2-way Switch
current to one of two routes according to its
(SPDT)
position. Some SPDT switches have a central
off position and are described as 'on-off-on'.

DPST = Double Pole, Single Throw.


Dual On-Off
A dual on-off switch which is often used to
Switch
switch mains electricity because it can isolate
(DPST)
both the live and neutral connections.

DPDT = Double Pole, Double Throw.


Reversing
This switch can be wired up as a reversing
Switch
switch for a motor. Some DPDT switches have
(DPDT)
a central off position.

An electrically operated switch, for example a


9V battery circuit connected to the coil can
Relay switch a 230V AC mains circuit.
NO = Normally Open, COM = Common,
NC = Normally Closed.

Resistors
Component Circuit Symbol Function of Component

A resistor restricts the flow of current, for


Resistor example to limit the current passing through an
LED. A resistor is used with a capacitor in a
timing circuit.
Some publications use the old resistor
symbol:

This type of variable resistor with 2 contacts (a


rheostat) is usually used to control current.
Variable Resistor Examples include: adjusting lamp brightness,
(Rheostat) adjusting motor speed, and adjusting the rate of
flow of charge into a capacitor in a timing
circuit.

This type of variable resistor with 3 contacts (a


potentiometer) is usually used to control
Variable Resistor
voltage. It can be used like this as a transducer
(Potentiometer)
converting position (angle of the control
spindle) to an electrical signal.

This type of variable resistor (a preset) is


operated with a small screwdriver or similar
tool. It is designed to be set when the circuit is
Variable Resistor
made and then left without further adjustment.
(Preset)
Presets are cheaper than normal variable
resistors so they are often used in projects to
reduce the cost.

Capacitors
Component Circuit Symbol Function of Component

A capacitor stores electric charge. A capacitor


is used with a resistor in a timing circuit. It
Capacitor
can also be used as a filter, to block DC
signals but pass AC signals.

A capacitor stores electric charge. This type


must be connected the correct way round. A
Capacitor,
capacitor is used with a resistor in a timing
polarised
circuit. It can also be used as a filter, to block
DC signals but pass AC signals.
Variable Capacitor A variable capacitor is used in a radio tuner.

This type of variable capacitor (a trimmer) is


Trimmer operated with a small screwdriver or similar
Capacitor tool. It is designed to be set when the circuit is
made and then left without further adjustment.

Diodes
Component Circuit Symbol Function of Component

A device which only allows current to flow


Diode
in one direction.

LED A transducer which converts electrical


Light Emitting Diode energy to light.

A special diode which is used to maintain a


Zener Diode
fixed voltage across its terminals.

Photodiode A light-sensitive diode.

Transistors
Component Circuit Symbol Function of Component

A transistor amplifies current. It can be used with other


Transistor NPN
components to make an amplifier or switching circuit.
A transistor amplifies current. It can be used with other
Transistor PNP
components to make an amplifier or switching circuit.

Phototransistor A light-sensitive transistor.

Audio and Radio Devices


Component Circuit Symbol Function of Component

A transducer which converts sound to electrical


Microphone
energy.

A transducer which converts electrical energy to


Earphone
sound.

A transducer which converts electrical energy to


Loudspeaker
sound.

A transducer which converts electrical energy to


Piezo Transducer
sound.

An amplifier circuit with one input. Really it is a block


Amplifier
diagram symbol because it represents a circuit rather
(general symbol)
than just one component.
Aerial A device which is designed to receive or transmit
(Antenna) radio signals. It is also known as an antenna.

Meters and Oscilloscope


Component Circuit Symbol Function of Component

A voltmeter is used to measure voltage.


Voltmeter The proper name for voltage is 'potential
difference', but most people prefer to say voltage!

Ammeter An ammeter is used to measure current.

A galvanometer is a very sensitive meter which is


Galvanometer
used to measure tiny currents, usually 1mA or less.

An ohmmeter is used to measure resistance. Most


Ohmmeter
multimeters have an ohmmeter setting.

An oscilloscope is used to display the shape of


Oscilloscope electrical signals and it can be used to measure
their voltage and time period.

Sensors (input devices)


Component Circuit Symbol Function of Component

A transducer which converts brightness (light) to


LDR resistance (an electrical property).
LDR = Light Dependent Resistor

A transducer which converts temperature (heat) to


Thermistor
resistance (an electrical property).
Logic Gates
Logic gates process signals which represent true (1, high, +Vs, on) or false (0, low, 0V, off).
For more information please see the Logic Gates page.
There are two sets of symbols: traditional and IEC (International Electrotechnical Commission).

Gate
Traditional Symbol IEC Symbol Function of Gate
Type

A NOT gate can only have one input. The


'o' on the output means 'not'. The output of
a NOT gate is the inverse (opposite) of its
NOT
input, so the output is true when the input
is false. A NOT gate is also called an
inverter.

An AND gate can have two or more inputs.


AND The output of an AND gate is true when all
its inputs are true.

A NAND gate can have two or more


inputs. The 'o' on the output means 'not'
NAND showing that it is a Not ANDgate. The
output of a NAND gate is true unless all its
inputs are true.

An OR gate can have two or more inputs.


OR The output of an OR gate is true when at
least one of its inputs is true.

A NOR gate can have two or more inputs.


The 'o' on the output means 'not' showing
NOR that it is a Not OR gate. The output of a
NOR gate is true when none of its inputs
are true.
An EX-OR gate can only have two inputs.
EX-OR The output of an EX-OR gate is true when
its inputs are different (one true, one false).

An EX-NOR gate can only have two


inputs. The 'o' on the output means 'not'
EX- showing that it is a Not EX-ORgate. The
NOR output of an EX-NOR gate is true when its
inputs are the same (both true or both
false).

MATTER
Everything in the world is made of matter. Matter is anything that has mass (weight) and
occupies space.

Matter can be made up of a group or series of different atoms to form a molecule. These groups
of atoms (molecules) are sometimes called compounds. Some types of matter can be broken
down to a single atom while still maintaining the properties of the original material. These types
of material are called elements.

Matter has three states: Solid, Liquid, and Vapor.


MOLECULE EXAMPLE
Imagine a lake. Now imagine taking the smallest particle or piece of water from the lake. You
would have a single molecule of water, H2O, which is made up of two hydrogen atoms and one
oxygen atom.

Not all materials are made up of molecules. Copper, for example, is made up of a single copper
atom. These are called elements. Each element is a type of matter that has certain individual
characteristics.

THE ATOM
One of the basic building blocks in the universe for matter is the
atom. All matter - gas, liquid, or solid - is made up of molecules
or atoms joined together. These atoms are the smallest particle
into which an element or substance can be divided without
losing its property.

A single atom consists of three basic components: a proton, a


neutron, and an electron.

Within the atom there is a Nucleus. The Nucleus contains the


protons and neutrons. Orbiting around the nucleus are the
electrons.

An atom is similar to a miniature solar system. As with the sun


in the center of the universe, the nucleus is in the center of the
atom. Protons and Neutrons are contained inside the nucleus.
Orbiting around the nucleus are the electrons.

ATOM CONSTRUCTION
An atom is similar to a miniature solar system. As the sun is in
the center of the solar system, so is the nucleus is in the center of
the atom. Protons and neutrons are contained within the nucleus.
Electrons orbit around the nucleus, which would be similar to
planets orbiting around the sun.

PROTONS
Protons are located within the nucleus of the atom (shown in
blue).

Protons are positively (+) charged.

NEUTRONS
Neutrons add atomic weight to an atom (shown in green).

Neutrons have no electrical charge.

ELECTRONS
Electrons orbit around the nucleus of the atom (shown in
yellow).

Electrons are negatively (-) charged.

Since electrons are lighter than protons and are outside the
nucleus, they can be easily moved from atom to atom to form a
flow of electrons. Normally electrons are prevented from being
pulled into the atom by the forward momentum of their rotation.
Electrons are also prevented from flying away because of the
magnetic attraction of the protons inside the nucleus, the same
type of force that keeps the planets orbiting around the sun.

ELECTRICAL CHARGES
Opposite electrical charges always attract each other. So these particles with opposite charges
will tend to move toward each other. Like electrical charges always repel. So particles with like
charges will move away from each other.

Remember: Opposites charges attract, and like charges repel.

Atoms always try to remain electrically balanced.

BALANCED ATOMS
Atoms normally have an equal number of electrons and protons.

Atoms have no electrical charge. They are neither positive nor


negative. They are electrically neutral or BALANCED.The
negative charge of the electrons will cancel the positive charge of
the protons, thus balancing the charge of the atom.

This cancellation of charges creates a natural attraction or bonding


between the positive proton and the negative electron.

ION PARTICLES
When an atom loses or gains an electron, an imbalance occurs.
The atom becomes either a positively or negatively charged
particle called an ION. These unbalanced charged ION particles
are responsible for electron flow (electricity).
IONs will take or release an electron to become balanced again.

ION CHARGE
A positive (+) ION has one less electron than it has protons.
A negative (-) ION has one more electron than it has protons.
The positive ION attracts a negative ION to become balanced. This
attraction or difference in electrical potential causes electron flow.

ELECTRON ORBITS
Electrons rotate around the atom at different orbits called Rings,
Orbits, or Shells.
BOUND ELECTRONS orbit the nucleus on the inner rings. Bound
electrons have a strong magnetic attraction to the nucleus.
FREE ELECTRONS orbit on the outermost ring which is known
as the VALANCE RING.

FREE ELECTRONS
Only the FREE ELECTRONS in the outermost shell (Valance
Ring) are free to move from atom to atom. This movement is
called ELECTRON FLOW.
These FREE ELECTRONS are loosely held and can easily be
moved to another atom or ion.
Because of their distance from the nucleus, free electrons have a
weak magnetic attraction. Since this attraction is not as strong to
the nucleus as the bound electrons on the inner orbits, the electrons
move easily from atom to atom.

INSULATORS
An INSULATOR is any material that inhibits (stops) the flow of
electrons (electricity).

An insulator is any material with 5 to 8 free electrons in the outer


ring.Because, atoms with 5 to 8 electrons in the outer ring are held
(bound) tightly to the atom, they CANNOT be easily moved to
another atom nor make room for more electrons.

Insulator material includes glass, rubber, and plastic.

CONDUCTORS
A CONDUCTOR is any material that easily allows electrons
(electricity) to flow.

A CONDUCTOR has 1 to 3 free electrons in the outer


ring.Because atoms with 1 to 3 electrons in the outer ring are
held (bound) loosely to the atom, they can easily move to
another atom or make room for more electrons.

Conductor material includes copper and gold.

SEMICONDUCTORS
Any material with exactly 4 free flectrons in the outer orbit are
called SEMICONDUCTORS.

A semiconductor is neither a conductor or insulator.

semiconductor material includes carbon, silicon, and germanium.

These materials are be used in the manufacturer of diodes,


transistors, and integrated circuit chips.
Two Current Flow theories exist. The first is:
ELECTRON THEORY
The Electron Theory states that current flows from NEGATIVE to POSITIVE.
Electrons move from atom to atom as they move through the conductor towards
positive.

The second Current Flow theory is:

CONVENTIONAL THEORY
Conventional theory, also known as HOLE THEORY, states that current flows from
POSITIVE to NEGATIVE. Protons or the lack of electrons (the holes) move towards
the negative. (Current flow direction in Hole Theory is the opposite of that in Electron
Theory.)

VOLTAGE
Voltage is the electrical force that moves electrons through a conductor. Voltage is
electrical pressure also known as EMF (Electro Motive Force) that pushes electrons.

The greater the difference in electrical potential push (difference between positive and
negative), the greater the voltage force potential.

MEASUREMENT
A VOLTMETER
measures the
voltage potential
across or parallel to
the circuit.

The Voltmeter
measures the
amount of electrical
pressure difference
between two points
being measured.

Voltage can exist


between two points
without electron
flow.

VOLTAGE UNITS
Voltage is measured in units called VOLTS.

Voltage measurements can use different value prefixes such


as millivolt, volt, Kilovolt, and Megavolt.

LARGER
LESS THAN
VOLTAGE BASIC UNIT THAN
BASE UNIT
BASE UNIT
Symbol mV V kV
Pronounced millivolt Volt Kilovolt
Multiplier 0.001 1 1,000

CURRENT (AMPERES)
CURRENT is the quantity or flow rate of electrons moving past a point within one second.
Current flow is also known as amperage, or amps for short.

Higher voltage will produce higher current flow, and lower voltage will produce lower current
flow.

MEASUREMENT
An AMMETER measures the quantity of current
flow.Ammeters are placed in series (inline) to count the
electrons passing through it.

Example: A water meter counts the gallons of water flowing


through it.

AMPERAGE UNITS
Current flow is measured in units called Amperes or AMPS.

Amperage measurements can use different value prefixes,


such as microamp, milliamp, and Amp.

LESS THAN LESS THAN


AMPERAGE BASIC UNIT
BASE UNIT BASE UNIT
Symbol A mA A
Pronounced Microamp milliamp Amp
Multiplier 0.000001 0.001 1

AFFECTS OF CURRENT FLOW


Two common effects of current flow are Heat Generation and Electromagnetism.

HEAT: When current flows, heat will be generated. The higher the current flow the greater the
heat generated. An example would be a light bulb. If enough current flows across the filament, it
will glow white hot and illuminate to produce light.

ELECTROMAGNETISM: When current flows, a small magnetic field is created. The higher the
current flow, the stronger the magnetic field. An example: Electromagnetism principles are used
in alternators, ignition systems, and other electronic devices.

RESISTANCE
Resistance is the force that reduces or stops the flow of
electrons. It opposes voltage.

Higher resistance will decrease the flow of electrons and


lower resistance will allow more electrons to flow.
MEASUREMENT
An OHMMETER measures the resistance of an electrical circuit or
component. No voltage can be applied while the ohmmeter is connected, or
damage to the meter will occur.

Example: Water flows through a garden hose, and someone steps on the
hose. The greater the pressure placed on the hose, the greater the hose
restriction and the less water flows.

RESISTANCE UNITS
Resistance is measured in units called OHMS.

Resistance measurements can use different value prefixes, such as Kilo ohm and Megaohms.

MORE MORE
AMPERAGE BASIC UNIT THAN THAN
BASE UNIT BASE UNIT
Symbol K M
Pronounced Ohm Kilo ohm Megaohm
Multiplier 1 1,000 1,000,000

RESISTANCE FACTORS
Various factors can affect the resistance. These include:

LENGTH of the conductor. The longer the conductor, the higher the resistance.

DIAMETER of the conductor. The narrower the conductor, the higher the resistance.

TEMPERATURE of the material. Depending on the material, most will increase resistance as
temperature increases.

PHYSICAL CONDITION (DAMAGE) to the material. Any damage will increase resistance.

TYPE of MATERIAL used. Various materials have a wide range of resistances.

TYPES OF ELECTRICITY
Two basic types of Electricity classifications:
STATIC ELECTRICITY is electricity that is standing still. Voltage potential with NOelectron
flow.

DYNAMIC ELECTRICITY is electricity that is in motion. Voltage potential WITH electron


flow. Two types of Dynamic electricity exist:

Direct Current (DC) Electron Flow is in only one direction.

Alternating Current (AC) Electron flow alternates and flows in both directions (back and
forth).

STATIC ELECTRICITY
Voltage potential with NO electron flow.

Example: By rubbing a silk cloth on a glass rod, you physically remove electrons from the glass
rod and place them on the cloth. The cloth now has a surplus of electrons (negatively charged),
and the rod now has a deficiency of electrons (positively charged).

Another example: Rub your shoes on a rug and then touch a metal table or chair .... Zap!! The
shock you felt was the static electricity dissipating through your body.

DYNAMIC ELECTRICITY
is electricity in motion, meaning you have electrons flowing, in
other words voltage potential WITH electron flow.

Two types of dynamic electricity exists:

Direct Current (DC)

Alternating Current (AC)

DIRECT CURRENT (DC)


Electricity with electrons flowing in only one direction is called Direct Current or
DC.

DC electrical systems are used in cars.


ALTERNATING CURRENT (AC)
Electricity with electrons flowing back and forth, negative - positive- negative, is called
Alternating Current, or AC.

The electrical appliances in your home use AC power.

SOURCES OF ELECTRICITY
Electricity can be created by several means: Friction, Heat, Light, Pressure, Chemical Action, or
Magnetic Action.

Only a few of these sources of energy are used in the automobile. The battery produces
electricity through chemical action, and the alternator produces electricity through magnetic
action.

Friction creates static electricity.


Heat can act upon a device called a thermo couple to create DC.
Light applied to photoelectric materials will produce DC electricity.
Pressure applied to a piezoelectric material will produce DC electricity.
Chemical Action of certain chemicals will create electricity.

Electrical Circuits
AN ELECTRICAL CIRCUIT
The circuit shown below has a power source, fuse, switch, two lamps and wires connecting each
into a loop or circle. When the connection is complete, current flows from the positive terminal
of the battery through the wire, the fuse, the switch, another wire, the lamps, a wire and to the
negative terminal of the battery. The route along which the electricity flows is called an electrical
circuit.

ELECTRICAL CIRCUIT REQUIREMENTS


A complete Electrical Circuit is required in order to make electricity practical. Electrons must
flow from and return to the power source.

There are three different circuit types, all require the same basic components:

1. Power Source is needed to supply the flow of electrons (electricity).


2. Protection Device prevents damage to the circuit in the event of a short.
3. Load Device converts the electricity into work.
4. Control Device allows the user control to turn the circuit on or off
5. Conductors provide an electrical path to and from the power source.

ELECTRICAL CIRCUIT REQUIREMENTS


A complete Electrical Circuit is required in order to make electricity practical. Electrons must
flow from and return to the power source.
There are three different circuit types, all require the same basic components:

1. Power Source is needed to supply the flow of electrons (electricity).


2. Protection Device prevents damage to the circuit in the event of a short.
3. Load Device converts the electricity into work.
4. Control Device allows the user control to turn the circuit on or off
5. Conductors provide an electrical path to and from the power source.

BASIC CIRCUIT CONSTRUCTION


1. Power Source (Battery, Alternator, Generator, etc.)
2. Protection Device (Fuse, Fusible Link, or Circuit Breaker)
3. Load Device (Lamp, Motor, Winding, Resistor, etc.
4. Control (Switch, Relay, or Transistor)
5. Conductors (A Return Path, Wiring to Ground)

LOADS
The illustration below has a horn in place of the lamp. Any device such as a lamp, horn, wiper
motor, or rear window defogger, that consumes electricity is called a load. In an electrical circuit,
all loads are regarded as resistance. Loads use up voltage and control the amount of current
flowing in a circuit. Loads with high resistance cause less current to flow while those with lower
resistance allow high current rates to flow.
AUTOMOTIVE ELECTRICAL CIRCUITS
In an automotive electrical circuit, one end of the wire from each load returning to the battery is
connected to the vehicle body or frame. Therefore, the vehicle body or frame itself functions as a
conductor, allowing current to flow though the body or frame and back to the battery. The body
or frame is then referred to as the body ground (or earth) of the circuit (meaning that part of the
circuit that returns the current to the battery).

WHAT IS OHM'S LAW?


A simple relationship exists between voltage, current, and resistance in electrical circuits.
Understanding this relationship is important for fast, accurate electrical problem diagnosis and
repair.

OHM'S LAW
Ohm's Law says: The current in a circuit is directly proportional to the applied voltage and
inversely proportional to the amount of resistance. This means that if the voltage goes up, the
current flow will go up, and vice versa. Also, as the resistance goes up, the current goes down,
and vice versa. Ohm's Law can be put to good use in electrical troubleshooting. But calculating
precise values for voltage, current, and resistance is not always practical ... nor, really needed. A
more practical, less time-consuming use of Ohm's Law would be to simply apply the concepts
involved:
SOURCE VOLTAGE is not affected by either current or resistance. It is either too low, normal,
or too high. If it is too low, current will be low. If it is normal, current will be high if resistance is
low, or current will be low if resistance is high. If voltage is too high, current will be high.

CURRENT is affected by either voltage or resistance. If the voltage is high or the resistance is
low, current will be high. If the voltage is low or the resistance is high, current will be low.

RESISTANCE is not affected by either voltage or current. It is either too low, okay, or too high.
If resistance is too low, current will be high at any voltage. If resistance is too high, current will
be low if voltage is okay.

NOTE: When the voltage stays the same, such as in an Automotive Circuit... current goes up as
resistance goes down, and current goes down as resistance goes up. Bypassed devices reduce
resistance, causing high current. Loose connections increase resistance, causing low current.

OHM'S LAW FORMULA


When voltage is applied to an electrical circuit, current flows in the circuit. The following special
relationship exists among the voltage, current and resistance within the circuit: the size of the
current that flows in a circuit varies in proportion to the voltage which is applied to the circuit,
and in inverse proportion to the resistance through which it must pass. This relationship is called
Ohm's law, and can be expressed as follows:

E=IR

Voltage = Current x Resistance

E Voltage applied to the circuit, in volts (V)

I Current flowing in the circuit, in amperes (A)

R Resistance in the circuit, in ohms

In practical terms "V = I x R" which means


"Voltage = Current x Resistance".

1 volt will push one amp through 1 ohm of resistance.

NOTE: E = IR, V=AR, or V=IR are all variations of the same formula. How you learned Ohm's
law will determine which one you will use. Personal preference is the only difference; anyone
will get you the correct answer.

OHM'S LAW SYMBOL SHORTCUT


Mathematical formulas can be difficult for many who don't use them regularly. Most people can
remember a picture easier than a mathematical formula. By using the Ohms law symbol below,
anyone can remember the correct formula to use. By knowing any two values you can figure out
the third. Simply put your finger over the portion of the symbol you are trying to figure out and
you have your formula.

APPLICATIONS OF OHM'S LAW


As an application of Ohm's law, any voltage V, current I or resistance R in an electrical circuit
can be determined without actually measuring it if the two others values are known.

This law can be used to determine the amount of current I flowing in the circuit when
voltage V is applied to resistance R. As stated previously, Ohm's law is:

Current = Voltage / Resistance.

In the following circuit, assume that resistance R is 2 and voltage V that is applied to it is 12 V.
Then, current I flowing in the circuit can be determined as follows:
This law can also be used to determine the voltage V that is needed to permit current I to pass
through resistance R: V = I x R (Voltage= Current x Resistance).

In the following circuit, assume that resistance R is 4 ohms. The voltage V that is necessary to
permit a current I of 3 A to pass through the resistance can be determined as follows:

Still another application of the law can be used to determine the resistance R when the voltage V
which is applied to the circuit and current I flowing in the circuit are already known:
In the following circuit, assume that a voltage V of 12 V is applied to the circuit and current I of
4 A flows in it. Then, the resistance value R of the resistance or load can be determined as
follows:

TYPES OF CIRCUITS
Individual electrical circuits normally combine one or more resistance or load devices. The
design of the automotive electrical circuit will determine which type of circuit is used. There are
three basic types of circuits:

Series Circuit

Parallel Circuit

Series-Parallel Circuit

SERIES CIRCUITS
A series circuit is the simplest circuit. The conductors, control and protection devices, loads, and
power source are connected with only one path to ground for current flow. The resistance of each
device can be different. The same amount of current will flow through each. The voltage across
each will be different. If the path is broken, no current flows and no part of the circuit works.
Christmas tree lights are a good example; when one light goes out the entire string stops
working.
SERIES CIRCUITS
A Series Circuit has only one path to ground, so electrons must go through each component to
get back to ground. All loads are placed in series.

Therefore:

1. An open in the circuit will disable the entire circuit.

2. The voltage divides (shared) between the loads.

3. The current flow is the same throughout the circuit.

4. The resistance of each load can be different.

SERIES CIRCUIT CALCULATIONS


If, for example, two or more lamps (resistances R1 and R2, etc.) are connected in a circuit as
follows, there is only one route that the current can take. This type of connection is called a
series connection. The value of current I is always the same at any point in a series circuit.
The combined resistance RO in this circuit is equal to the sum of individual resistance R1 and
R2. In other words: The total resistance(RO) is equal to the sum of all resistances (R1 + R2 + R3
+ .......)

Therefore, the strength of current (I) flowing in the circuit can be found as follows:

Resistance R0 (a combination of resistances R1 and R2, which are connected in series in the
circuit as illustrated) and current I flowing in this circuit can be determined as follows:
VOLTAGE DROP
A voltage drop is the amount of voltage or electrical pressure that is used or given up as electrons
pass through a resistance (load). All voltage will be used up in the circuit. The sum of the voltage
drops will equal source voltage. A voltage drop measurement is done by measuring the voltage
before entering the load and the voltage as it leaves the load. The difference between these two
voltage readings is the voltage drop.

VOLTAGE DROP TOTAL


When more than one load exists in a circuit, the voltage divides and will be shared among the
loads. The sum of the voltage drops equal source voltage. The higher the resistance the higher
the voltage drop. Depending on the resistance, each load will have a different voltage drop.

0V + 5V + 7V + 0V = 12V
VOLTAGE DROP CALCULATION
When current flows in a circuit, the presence of a resistance in that circuit will cause the voltage
to fall or drop as it passes through the resistance. The resultant difference in the voltage on each
side of the resistance is called a voltage drop. When current (I) flows in the following circuit,
voltage drops V1 and V2 across resistances R1 and R2 can be determined as follows from Ohm's
law. (The value of current I is the same for both R1 and R2 since they are connected in series.)

The sum of the voltage drops across all resistances is equal to the voltage of the power source
(VT):

The voltage drop across resistances R1 and R2 in the following circuit can be determined as
follows:
PARALLEL CIRCUIT
A parallel circuit has more than one path for current flow. The same voltage is applied across
each branch. If the load resistance in each branch is the same, the current in each branch will be
the same. If the load resistance in each branch is different, the current in each branch will be
different. If one branch is broken, current will continue flowing to the other branches.
PARALLEL CIRCUIT
In parallel connection, two or more resistances (R1, R2, etc.) are connected in a circuit as
follows, with one end of each resistance connected to the high (positive) side of the circuit, and
one end connected to the low (negative) side. Full battery voltage is applied to all resistances
within a circuit having a parallel connection.
Resistance R0 (a combination of resistances R1 and R2) in a parallel connection can be
determined as follows:

From the above, the total current I flowing in this circuit can be determined from Ohm's law as
follows:
The total current I is also equal to the sum of currents I1 and I2 flowing through individual
resistances R1 and R2

Since battery voltage V is applied equally to all resistances, the strength of currents I1 and I2 can
be determined from Ohm's law as follows:

Resistance RO (a combination of resistances R1 and R2, which are connected in parallel in the
circuit as shown below), the total current I flowing in the circuit, and currents I1 andI2 flowing
through resistances R1 and R2, can be determined respectively as follows:
SERIES PARALLEL CIRCUIT
A series-parallel circuit has some components in series and others in parallel. The power source
and control or protection devices are usually in series; the loads are usually in parallel. The same
current flows in the series portion, different currents in the parallel portion. The same voltage is
applied to parallel devices, different voltages to series devices. If the series portion is broken,
current stops flowing in the entire circuit. If a parallel branch is broken, current continues
flowing in the series portion and the remaining branches.
SERIES-PARALLEL CIRCUIT
A resistance and lamps may be connected in a circuit as illustrated below. This type of
connecting method is called series-parallel connection, and is a combination of series and
parallel connections. The interior dash board lights are a good example. By adjusting the
rheostat, you can increase or decrease the brilliance of the lights.
The combined resistance R02 in this series-parallel connection can be determined in the
following order:

a. Determine combined resistance R01, which is a combination of resistances R2 and R3


connected in parallel.

b. Then, determine resistance R02, which is a combination of resistance R1 and combined


resistance R01 connected in series.
Total current I flowing in the circuit can be determined from Ohm's law as follows:

The voltage applied to R2 and R3 can be found by the following formula:

Currents I1, I2 and I flowing through resistances R1, R2 and R3 in the series-parallel
connection, as shown below, can be determined as follows:
CONTROL DEVICES
CONTROLS

Control devices are used to "turn on" or "turn off" current flow in an electrical circuit.
Control devices include a variety of switches, relays, and solenoids. Electronic control
devices include capacitors, diodes, and transistors, but those will be discussed in
another training module.

Control devices are needed to start, stop, or redirect current flow in an electrical circuit.
Most switches require physical movement for operation while relays and solenoids are
operated with electromagnetism.

CONTROL DEVICES

SWITCHES

Single Pole Single Throw (SPST)


Single Pole Double Throw (SPDT)
Momentary Contact
Multiple Pole Multiple Throw (MPMT or Gang Switch)
Mercury
Temperature (Bimetal)
Time Delay
Flasher

RELAYS

SOLENOIDS

SWITCHES
A switch is the most common circuit control device. Switches usually have two or more sets
of contacts. Opening these contacts is called "break" or "open" the circuit, Closing the
contacts is called "make" or "completing" the circuit.

Switches are described by the number of Poles and Throws they have. "Poles" refer to the
number of input circuit terminals while "Throws" refer to the number of output circuit
terminal. Switches are referred to as SPST (single-pole, single-throw), SPDT (single-pole,
double-throw), or MPMT (multiple-pole, multiple-throw).

SINGLE POLE SINGLE THROW (SPST)

The simplest type of switch is a "hinged pawl" or "knife blade" switch. It either "completes"
(turn on) or "break" (turn off) the circuit in a single circuit. This switch has a single input
pole and a single output throw.

SINGLE POLE DOUBLE THROW (SPDT)

A single-pole input, double-throw output switch has one wire going it and two wires coming
out. A Headlamp dimmer switch is a good example of a single-pole double-throw switch.
The switch sends current to either the high-beams or low-beams of the headlight circuit.

MULTIPLE POLE MULTIPLE THROW (MPMT)


Multiple-Pole input, Multiple-Throw output switches, which are also known as "gang"
switches, have movable contacts in wired in parallel. These switches move together to
supply different sets of output contacts with current. An ignition switch is a good example of
a multiple-pole multiple-throw switch. Each switch sends current from different source to
different output circuits at the same time depending on position. The dotted line between
the switches indicates they move together; one will not move without the other moving as
well.

MOMENTARY CONTACT
The momentary contact switch has a spring-loaded contact that keeps it from making the
circuit except when pressure is applied to the button. This is a "normally open" type
(shown below). A horn switch is a good example of a momentary contact switch. Push the
horn button and the hold sounds; release the button and the horn stops.

A variation of this type is the normally closed (not shown) which works the opposite as
described above. The spring holds the contacts closed except when the button is pressed. In
other words the circuit is "ON" until the button is pushed to break the circuit.

MERCURY
A mercury switch is made of a sealed capsule that is partially filled with mercury. In one end
of the capsule are two electrical contacts. As the switch is rotated (moved from true
vertical) the mercury flows to the opposite end of the capsule with the contacts, completing
the circuit. Mercury switches are often be used to detect motion, such as the one used in
the engine compartment on the light. Other uses include fuel cut off for roll-overs, and
some air bag sensor applications. Mercury is a hazardous waste and should be handled with
care.

BI-METALLIC
A temperature-sensitive switch, also known as a "bi-metallic" switch, usually contains a
bimetal element that bends when heated to make contact completing a circuit or to break
contact opening a circuit. In an engine coolant temperature switch, when the coolant
reaches the temperature limit, the bimetal element bends causing the contacts in the switch
to close. This completes the circuit and lights the warning indicator on the instrument panel.

TIME DELAY SWITCH

The time delay switch contains a bimetal strip, contacts, and a heating element. The time
delay switch is normally closed. As current flows through the switch, current flows through
the heating element causing it to heat, which causes the bimetal strip to bend and open the
contacts. As current continues to flows through the heating element, the bimetal strip is
kept hot, keeping the switch contacts open. The amount of time delay before the contacts
open is determined by the characteristics of the bimetal strip and the amount of heat
produced by the heating element. When power to the switch is turned off, the heating
element cools and the bimetal strip returns to the rest position and the contacts are closed.
A common application for a time delay switch is the rear window defroster.

FLASHER
The flasher operates basically the same as the time delay switch; except when the contacts
open, current stops flowing through the heating element. This causes the heating element
and bimetal strip to cool. The bimetal strip returns to the rest position which closes the
contacts, allowing current to flow through the contacts and heating element again. This
cycle repeats over and over until power to the flasher is eliminated. Common uses for this
type of switch are the turn signals or the four-way flasher (hazard lamps).

RELAYS
A relay is simply a remote-control switch, which uses a small amount of current to control a
large amount of current. A typical relay has both a control circuit and a power circuit.
Relay construction contains an iron core, electromagnetic coil, and an armature (moveable
contact set). There are two types of relays: normally open (shown below) and normally
closed (NOT shown). A Normally open (N.O.) relay has contacts that are "open" until the
relay is energized while a normally closed (N.C.) relay has contacts that are "closed" until
the relay is energized.
RELAY OPERATION

Current flows through the control coil, which is wrapped around an iron core. The iron core
intensifies the magnetic field. The magnetic field attracts the upper contact arm and pulls it
down, closing the contacts and allowing power from the power source to go to the
load. When the coil is not energized, the contacts are open, and no power goes to the load.
When the control circuit switch is closed, however, current flows to the relay and energizes
the coil. The resulting magnetic field pulls the armature down, closing the contacts and
allowing power to the load. Many relays are used for controlling high current in one circuit
with low current in another circuit. An example would be a computer, which controls a relay,
and the relay controls a higher current circuit.

SOLENOIDS - PULLING TYPE

A solenoid is an electromagnetic switch that converts current flow into mechanical


movement. As current flows through the winding a magnetic field is created. The magnetic
field will pull the moveable iron core into the center of the winding. This type of solenoid is
called a "pulling" type solenoid, as the magnetic field pulls the moveable iron core into the
coil. A common use for pulling solenoids are in the starting system. The starter solenoid
engages the starter with the flywheel.
SOLENOIDS - PULLING TYPE OPERATION

As current flows through the winding a magnetic field is created. These magnetic lines of
force want to be as small as possible. If an iron core is placed near the coil that has current
flowing through it, the magnetic field will stretch out like a rubber band, reaching out and
pulling the iron bar into the center of the coil.

SOLENOIDS - PUSH/PULL TYPE

In a "push-pull" type solenoid, a permanent magnet is used for the core. Since "like"
magnetic charges repel and "unlike" magnetic charges attract, by changing the direction of
current flow through the coil, the core is either "pulled in" or "pushed out." A common use
for this type of solenoid is on electric door locks.

CIRCUIT PROTECTION
CIRCUIT PROTECTION
Circuit protection devices are used to protect wires and connectors from being damaged by
excess current flow caused by either an over current or short-circuit. Excess current causes
excess heat, which causes circuit protection to "open circuit".

CIRCUIT PROTECTION DEVICES


Fuses, fuse elements, fusible links, and circuit breakers are used as circuit protection
devices. Circuit protection devices are available in a variety of types, shapes, and specific
current ratings.
FUSES
A fuse is the most common protection device. A fuse is placed in an electrical circuit, so that
when current flow exceeds the rating of the fuse it "blows" or "blows out". The element in
the fuse melts, opening the circuit and preventing other components of the circuit from
being damaged by the overcurrent. The size of the metal fuse element determines its
rating. Remember, excessive current causes excess heat, and it's the heat and not the
current that causes the circuit protector to open. Once a fuse "blows" it must be replaced
with a new one.

FUSE LOCATIONS
Fuses are located throughout the entire vehicle. Common locations include the engine
compartment, behind the left or right kick panels, or under the dash. Fuses are usually
grouped together and are often mixed in with other components like relays, circuit breakers,
and fuse elements.

FUSE BLOCK COVERS


Fuse / relay block covers usually label the location and position of each fuse, relay, and fuse
element contained within.
FUSE TYPES
Fuses are classified into basic categories: blade type fuses or cartridge type fuses. Several
variations of each are used.

COMMON FUSE TYPES


The blade fuse and fuse element are by far the most commonly used today. Three different
types of blade fuses exist; The Maxi Fuse, The Standard Auto fuse, and the Mini fuse. The
fuse element has replaced the fusible link and will be explained later.
BASIC CONSTRUCTION
The blade type fuse is a compact design with a metal element and transparent insulating
housing which is color-coded for each current rating. (Standard Auto shown below; however
construction of both the mini and maxi fuses are the same.)
FUSE AMPERAGE COLOR RATING
Fuse amperage color ratings for both the mini and standard ATO fuses are identical.
However, the amperage color ratings of maxi fuses use a different color scheme.

Color Ratings For STANDARD and MINI Fuses


Fuse Amp Rating Identification Color
3 Violet
5 Tan
7.5 Brown
10 Red
15 Blue
20 Yellow
25 Colorless
30 Green

MAXI STANDARD MINI

Color Ratings For MAXI Fuses


Fuse Amp Rating Identification Color
20 Yellow
30 Green
40 Amber
50 Red
60 Blue
70 Brown
80 Colorless

OLDER TYPE FUSES


Many older vehicles, both foreign and domestic, use glass or ceramic fuse cartridges that
were either color coded or stamped on case for current ratings. Glass fuses were used on
older domestic vehicles while the ceramic were used on most older European vehicles.
Ceramic fuses used an amperage color rating system while glass fuses have the amperage
ratings stamped into one of the metal end caps.

FUSIBLE LINKS AND FUSE ELEMENTS


Fusible links are divided into two categories: the fuse element cartridge and the fusible link.
The construction and function of fusible links and fuse elements are similar to that of a fuse.
The main difference is that the fusible link and fuse element are used to protect higher
amperage electrical circuits, generally circuits 30 amps or more. As with fuses, once a
fusible link or fuse element blows out, it must be replaced with a new one.

FUSE ELEMENT CARTRIDGE


Fuse elements, a cartridge type fusible link, are also known as a Pacific fuses. The element
has the terminal and fusing portion as a unit. Fuse elements have replaced fusible links for
the most part. The housing is color coded for each current rating. Although fuse elements
are available in two physical sizes and are either plug in or bolt on design, the plug-in type
is the most popular.
FUSE ELEMENT CARTRIDGE CONSTRUCTION
Construction of the fuse element is quite simple. A colored plastic housing contains the
fusing portion element which can be viewed through a clear top. Fuse ratings are also
stamped on the case.

FUSE ELEMENT COLOR IDENTIFICATION


Fuse amperage color ratings are shown below. The fusing portion of the fuse element is
visible through a clear window. The amperage ratings are also listed on the fuse element.
Fuse Element Color Ratings - Pacific
Amperage Rating Identification Color
30 Pink
40 Green
50 Red
60 Yellow
80 Black
100 Blue

FUSIBLE ELEMENTS
Fusible elements are often located near the battery by themselves.

FUSIBLE ELEMENTS
Fusible elements can also be located in relay / fuse boxes in the engine compartment.
FUSIBLE LINKS
Fusible links are short pieces of a smaller diameter wire designed to melt during an over
current condition. A fusible link is usually four (4) wire sizes smaller than the circuit that it is
protecting. The insulation of a fusible link is a special nonflammable material. This allows
the wire to melt, but the insulation to remain intact for safety. Some fusible links have a tag
at one end that indicates its rating. Like fuses, fusible links must be replaced after they
have "blown" or melted opened. Many manufacturers have replaced fusible links with fuse
elements or maxi fuses.

CIRCUIT BREAKERS
Circuit breakers are used in place of fuses for the protection of complicated power circuits
such as the power windows, sunroofs and heater circuits. Three types of circuit breakers
exists: The manual reset type - mechanical, the automatic resetting type - mechanical, and
the automatically reset solid state type - PTC. Circuit breakers are usually located in
relay/fuse boxes; however, some components like power window motors have circuit
breakers built in.
CIRCUIT BREAKER CONSTRUCTION (MANUAL TYPE)
A circuit breaker basically consists of a bimetal strip connected to two terminals and to a
contact in between. Manual circuit breaker when tripped (current flow beyond its rating) will
open and must be reset manually. These manual circuit breakers are called "non-cycling"
circuit breakers.

CIRCUIT BREAKER OPERATION (MANUAL TYPE)


The circuit breaker contains a metal strip made of two different metals bonded together
called a bimetal strip. This strip is in the shape of a disc and is concaved downward. When
heat from the excessive current is higher than the circuit breaker current rating, the two
metals change shape unevenly. The strip bends or warps upwards and the contacts open to
stop current flow. The circuit breaker can be reset after it is tripped.
MANUAL RESET TYPE
When a circuit breaker is opened by an over-current condition, the circuit breaker requires
reset. To do so, insert a small rod (paper clip) to reset the bimetal plate as shown below.

AUTOMATIC RESETTING TYPE - MECHANICAL


Circuit breakers that automatically reset are called "cycling" circuit breakers. This type of
circuit breaker is used to protect high current circuits, such as power door locks, power
windows, air conditioning, etc. The automatically resetting circuit breaker contains a bimetal
strip. The bimetal strip will overheat and open from the excess current by an overcurrent
condition and is automatically reset when the temperature of the bimetal strip cools.
AUTO RESET CONSTRUCTION AND OPERATION
A cycling circuit breaker contains a metal strip made of two different metals bonded
together called a bimetal strip. When heat from the excessive current is higher than the
circuit breaker current rating the two metals change shape unevenly. The strip bends
upwards and a set of contacts open to stop current flow. With no current flowing the bimetal
strip cools and returns to its normal shape, closing the contacts, and resuming the current
flow. Automatically resetting circuit breakers are said to "cycle" because they cycle open
and closed until the current returns to a normal level.

AUTOMATIC RESETTING SOLID STATE TYPE - PTC


A Polymer PTC (for Positive Temperature Coefficient) is a special type of circuit breaker
called a thermistor (or thermal resistor). A PTC thermistor increases resistance as its
temperature in increased. PTCs, which are made from a conductive polymer, are solid state
devices, which means they have no moving parts. PTCs are commonly used to protect
power window and power door lock circuits.
POLYMER PTC CONSTRUCTION AND OPERATION
In its normal state, the material in a polymer PTC is in the form of a dense crystal, with
many carbon particles packed together. The carbon particles provide conductive pathways
for current flow. This resistance is low. When the material is heated from excessive current,
the polymer expands, pulling the carbon chains apart. In this expanded "tripped" state,
there are few pathways for current. When current flow exceeds the trip threshold, the
device remains in the "open circuit" state as long as voltage remains applied to the circuit.
It resets only when voltage is removed and the polymer cools. PTCs are used to protect
power window and power door lock circuits.

VARIABLE RESISTOR

RESISTORS
All electrical circuits require resistance to operate correctly. Resistors are sometimes added
to an electrical circuit to limit current flow, create voltage drops, or provide different
operating modes. All resistors are rated in both a fixed ohm value of resistance and a power
rating in watts. (Watt = Volts X Amps)

Three basic categories of resistors are used in automotive electrical systems:


1. Fixed;

2. Stepped or tapped;

3. Variable.

Each has different characteristics and usage ranging from a simple fan circuit to a
completed computer circuit.

FIXED RESISTORS

Fixed-value resistors are divided into two category types of resistors: Carbon / Metal
Oxide and Wire-Wound.

Carbon and Metal Oxide Film Wire-Wound

Fixed Resistor
Electrical Symbol

CARBON RESISTORS
Carbon resistors are commonly used in electronic systems. Carbon is mixed with binder; the
more carbon, the lower the resistance. Carbon resistors have a fixed resistance value and
are used to limit current flow. They are rated in watts and most have color-code bands to
show the resistance value. A typical resistor has a watt rating from 0.125W to 2.0 W.

Note: Metal-Oxide Film is sometimes used instead of carbon. While carbon is commonly
used for ratings up to 0.5 watt , Metal-Oxide Film provide, better high-temperature
satiability and is often used for 1.0 - 2.0 watt resistors.
Carbon Metal Oxide Film

RESISTOR RATING COLOR BANDS

The first two bands set the digit or number value of the resistor.

The third band, also known as the multiplier band, is the number of zeros added to the
number value.

The last band is the Tolerance band. Example: +/- 10%

RESISTOR COLOR BAND CHART


The chart below is used to interpret the color bands on the carbon resistor. Another chart is
used to show the value of tolerance band colors (not shown).

READING COLOR BANDS - RATING VALUE

Using the illustration below:


The first color band is Green with a value of "5".
The second color band is Red with a value of "2".
The third band is Black with a value of "0" zero. (No zeros are added)

So the resistor has a base value of 52 ohms.

READING COLOR BANDS - TOLERANCE VALUE


Resistors vary in tolerance (accuracy). Common tolerance values are 20%, 10%, 5%, 2%,
or 1%, simply meaning the maximum percent allowable difference the resistor value
actually is from the designed value rating. A 1% resistor is a higher quality resistor than one
with a 20% rating.

The tolerance band (last band) is silver with a value of 10%.

So, the resistance value is "52 ohms plus or minus 5.2 ohms" (46.8
to 57.2 ohms).

FIXED RESISTOR USAGE


Fixed resistors are often used in voltage division circuits. One example is the computer
sensor circuit shown below. Notice two resistors, R1 and R2, are placed in series. A fixed
resistor R1, known as a pull-up resistor, is used to create a voltage drop point. As the
resistance of water temp sensor (R2) changes, so does its voltage drop. This change also
changes the voltage drop of R1. A monitor circuit inside the computer measures this voltage
drop in between the two resistors.
WIRE-WOUND RESISTORS
Wire-wound resistors are made with coils of resistance wire. Often enclosed in ceramic to
help dissipate heat and protect the resistor wire, they are accurate and heat stable. The
resistance value is often marked. Wire-wound are used in higher watt circuits often 2W or
higher. An ignition ballast resistor is an example of a wire wound resistor.

STEPPED OR TAPPED RESISTORS


A stepped or tapped resistor has two or more fixed taps that provide different resistance
values. These taps allow current to flow through all or part of the resistor, which changes
the amount of current flowing through the circuit. Stepped resistors can also be encased in
ceramic and are nothing more than a series of fixed resistors placed end to end.
STEPPED RESISTOR OPERATION
An example of a stepped resistor in operation is the blower motor circuit shown below.
Notice the blower resistor is in series with the blower motor. Higher resistance in series will
lower the current flow; thus, higher blower resistor resistance will result in lower blower
speeds.

In low speed current flows through the entire resister from pin#3 though pin#1 to ground
(shown in blue). The motor will be in low speed because less current is flowing. Moving the
blower switch to medium 2 speed opens a path for current out pin #4 to ground. Current
flows through pin#3 and out pin#4 to ground (shown in green). Current will flow through
only part of the resistor. The motor will be in medium 2 speed causing the blower motor to
spin faster compared to low speed because more current is flowing through the circuit.

VARIABLE RESISTORS
Variable resistors provide an infinite amount of resistance values. Variable resistors are
used by electrical circuits to provide information on temperature, position, or light source.
There are four types of variable resistors used:

Rheostats

Potentiometers

Thermistors
Generic Variable Resistor
Photoresistor Electrical Symbol

RHEOSTAT
Rheostats at one time were used in the headlamp switch to dim or brighten dash panel
lighting and pre-OBDII fuel gauge sending units. Rheostats have two connections, one to
the fixed end of a resistor and the other to a sliding contact on the resistor. Turning the
control moves the sliding contact away from or toward the fixed end, increasing or
decreasing the resistance. Rheostats control resistance, thus controlling current flow.

Rheostat Symbol Variable Resistor Symbol

RHEOSTAT OPERATION
As the wiper moves along the rheostat it exposes more or less of the resistor. Moving the
wiper towards the high places a small portion of the resistor in series with the light, causing
the light to glow bright. Moving the wiper toward the low, places a larger portion of the
resistor in series with the lamp; this increased resistance causes less current to flow
lowering the intensity of the light. Rheostats are not used on computer circuits because of
temperature variations on the resistor when the wiper arm is moved.
POTENTIOMETER
Potentiometers are used to measure changes in position. Potentiometers have three
connections or legs: the reference, signal, and ground. The reference is at one end of a
resistor and the Ground is at the other end. Current flows from the Reference through the
resistor to Ground creating a voltage drop across the resistor. The Signal is a sliding
contact (movable wiper arm) that runs across the resistor. Unlike a rheostat, its main
purpose is not to vary resistance but to vary the voltage in a circuit.

Potentiometer Variable Resistor


Symbol Symbol

POTENTIOMETER OPERATION
Remember a potentiometer has three legs, the reference (R), the signal (S) , and the
ground (G) as shown below. 5 volts is supplied to the reference, current flows from the
reference (R) through the entire resistor to ground (G). The Signal wiper slides across the
resistor changing measure voltage as it moves. As the wiper moves towards the reference
(R), the measured signal voltage at (S) will increase. As the wiper moves away from the
Reference (R) towards ground (G), the measured signal voltage drops.
POTENTIOMETER APPLICATIONS
Since potentiometer are used to measure changes in position they naturally are used for
throttle, EGR, AC blend door, and power seat position sensors. All potentiometers have
three wires and are used to measure position changes.

THERMISTOR
Thermistors are resistors that change resistance as the temperature changes. These are
ideal in electrical circuits where measuring temperature change is required. There are two
types of thermistors:

NTC (Negative Temperature Coefficient)


NTC thermistors: as the temperature goes down, the resistance goes up, and as the
temperature goes up, the resistance drops. This type is used today in automotive
applications.

PTC (Positive Temperature Coefficient)


PTC thermistors: as the temperature go down, the resistance goes down also, and as the
temperature goes up, the resistance increases.
Thermistor
Electrical Symbol

THERMISTOR APPLICATIONS
Thermistors are used as Air, Coolant, EGR, and Automatic Air Temperature sensors. All
thermistors have two wires and are used to measure temperature changes. When placed in
series with a fixed (pull-up) resistor, thermistors create a variable voltage drop circuit,
which is ideal for use by computer circuits. Engine coolant temperature sensor (ECT) is
shown below.

THERMISTOR OPERATION
In the example below, as the engine warms up, R2 (water temp sensor) increases in
temperature; this lowers the resistance of R2. The drop in R2 resistance results in a
decreased voltage drop across R2 (less voltage is used). This decreased voltage drop across
R2 causes the voltage drop across R1 to increase. R1 now uses more voltage. This increased
voltage drop of R1 results in a lowered measured voltage by the monitor circuit and is seen
by the computer as an increase in engine temperature. The reverse is true as the water
temp sensor cools.
PHOTORESISTORS
A photoresistor, or photoconductive cell, is basically a light sensitive resistor whose
resistance changes as light is exposed to it. Basically photoresistors change resistance as
light intensity changes. The photoresistor has high resistance with no light exposed and
decreases in resistance as light intensity increases. A common automotive use for
photoresistors is the automatic headlamp circuit.

Photo Resistor
Electrical Symbol

PHOTORESISTOR OPERATION
A photoresistor is placed in the automatic headlight circuit shown below. The photoresistor
operates a normally closed headlight relay. During daylight driving light shines on the
photoresistor, lowering its resistance, causing current to flow allowing the relay coil to
energize, opening the relay contacts, thus preventing the headlights from operating. When
the photoresistor is in darkness, its resistance increases preventing enough current flow
through the relay coil to keep the relay contacts open. The closed relay contacts allow
current to flow to the headlights causing them to turn on. Photoresistors respond slowly to
changes in light intensity and may require several minutes to stabilize.
RELAYS

RELAYS
Relays are used throughout the automobile. Relays which come in assorted sizes, ratings,
and applications, are used as remote control switches. A typical vehicle can have 20 relays
or more.
RELAY LOCATIONS
Relays are located throughout the entire vehicle. Relay blocks, both large and small, are
located in the engine compartment; behind the left or right kick panels, or under the dash
are common locations. Relays are often grouped together or with other components like
fuses or placed by themselves.

RELAY POSITION IDENTIFICATION


Relay / Fuse block covers usually label the location and position of each fuse, relay, or fuse
element contained within.

RELAY APPLICATIONS
Relays are remote control electrical switches that are controlled by another switch, such as
a horn switch or a computer as in a power train control module. Relays allow a small current
flow circuit to control a higher current circuit. Several designs of relays are in use today, 3-
pin, 4-pin, 5-pin, and 6-pin, single switch or dual switches.
RELAY OPERATION
All relays operate using the same basic principle. Our example will use a commonly used 4 -
pin relay. Relays have two circuits: A control circuit (shown in GREEN) and a load circuit
(shown in RED). The control circuit has a small control coil while the load circuit has a
switch. The coil controls the operation of the switch.

RELAY ENERGIZED (ON)


Current flowing through the control circuit coil (pins 1 and 3) creates a small magnetic field
which causes the switch to close, pins 2 and 4. The switch, which is part of the load circuit,
is used to control an electrical circuit that may connect to it. Current now flows through pins
2 and 4 shown in RED, when the relay is energized.

RELAY DE-ENERGIZED (OFF)


When current stops flowing through the control circuit, pins 1 and 3, the relay becomes de-
energized. Without the magnetic field, the switch opens and current is prevented from
flowing through pins 2 and 4. The relay is now OFF.

RELAY OPERATION
When no voltage is applied to pin 1, there is no current flow through the coil. No current
means no magnetic field is developed, and the switch is open. When voltage is supplied to
pin 1, current flow though the coil creates the magnetic field needed to close the switch
allowing continuity between pins 2 and 4.
NORMALLY DESIGN ID
Relays are either Normally Open or Normally Closed. Notice the position of the switches in
the two relays shown below. Normally open relays have a switch that remains open until
energized (ON) while normally closed relays are closed until energized. Relays are always
shown in the de-energized position (no current flowing through the control circuit - OFF).
Normally open relays are the most common in vehicles; however either can be use in
automotive applications.

Normally Open (NO) Normally Closed (NC)

NORMALLY CLOSED RELAYS


The operation of a Normally Closed relay is the same to that of a Normally Open relay,
except backwards. In other words, when the relay control coil is NOT energized, the relay
switch contacts are closed, completing the circuit through pins 2 and 4. When the control
coil is energized, the relay switch contacts opens, which breaks the circuit open and no
continuity exists between pins 2 and 4.
DE - ENERGIZED (OFF) ENERGIZED (ON)

ACTUAL RELAY DESIGN


Current flows through the control coil, which is wrapped around an iron core. The iron core
intensifies the magnetic field. The magnetic field attracts the upper contact arm and pulls it
down, closing the contacts and allowing power from the power source to go to the load.
RELAY VARIATIONS
Other relay variations include three and five pin relays. A 3-PIN relay instead of two B+
input sources, this relay has one B+ input at pin 1. Current splits inside the relay, supplying
power to both the control and load circuits. A 5-PIN relay has a single control circuit, but
two separate current paths for the switch: One when the relay is de-energized (OFF - no
current through the control coil) and the other the energized (ON - current is flowing
through the control coil). When the 5-PIN relay is de-energized (OFF), pins 4 and 5 have
continuity. When the relay is energized (ON), pins 3 and 5 have continuity.

3 - PIN 4 - PIN 5 - PIN


ISO STANDARDIZED RELAYS
ISO relays were designed to try and standardize relay connections, making it easier to test
and design systems. ISO relays are currently used by almost all automotive manufacturers
today. Both 4 and 5 pin designs are used in both standard mini and micro sizes. FYI: ISO is
short for International Standard Organization.

STANDARD MINI SHOWN

STANDARD MINI ISO RELAYS TYPES


Below are two popular standard MINI ISO relay configurations. The size of a ISO Standard
MINI relay is a 1" square cube. Both 4 and 5 pins designs are used.
5 PIN
MINI RELAY

4 PIN
MINI RELAY

ISO MICRO RELAY TYPES


Below are two popular MICRO ISO relay configurations. The size of a ISO MICRO relay is a
1" x 1" x 1/2" square (1/2 as thick as a Mini relay). Both 4 and 5 pins designs are used.

5 PIN
MICRO RELAY
4 PIN
MICRO RELAY

VOLTAGE SPIKES
When the switch is closed (shown left), current flows through the coil from positive to
negative as shown in red. This current flow creates a magnetic field around the coil. The top
of the coil is positive, and the bottom is negative.

When the switch is opened (shown on right), current stops flowing through the control
circuit coil, and the magnetic field surrounding the coil cannot be maintained. As the
magnetic field collapses across the coil, it induces a voltage into itself, creating a reverse
polarity voltage spike of several hundred volts. Although the top of the coil is still 12 volts
positive, the bottom of the coil produces several hundred positive volts (200+ volts or
more); 200 is "more positive" and stronger than 12 volts, so current flows from the bottom
of the coil up towards the top.

VOLTAGE SUPPRESSION RELAYS


Relays are often controlled by a computer. When relays are controlled by semiconductors
such as transistors, they require some type of voltage suppression device. Solid state
circuits are vulnerable to voltage spikes. Voltage spikes slam against transistors, destroying
them. While some computer circuits have voltage suppression built inside the computer,
others rely on voltage suppression from within the relay. High ohm resistors, diodes, or
capacitors can be used for voltage suppression. Diodes and resistors are the most common.
NOTE: Relays are usually clearly marked if a suppression diode or resistor are present.

RELAYS WITH DE-SPIKING DIODES


A de-spiking (clamping) diode is connected in parallel with the relay coil. It is in the reverse
biased position when the relay is turned on; therefore no current will flow through the
diode. When the relay control circuit is opened (turned OFF), current stops flowing through
the coil, causing the magnetic field to collapse. The magnetic lines of force cut through the
coil and induce a counter voltage (a voltage in reverse polarity) into the winding. The
counter voltage begins to raise. When the bottom side of the diode sees .7 volts more
positive voltage than the top, the diode becomes forward biased, allowing the excess
voltage to pass, completing the circuit to the other end of the coil. The current flows around
in the diode and coil circuit until the voltage is dissipated.

RELAYS WITH DE-SPIKING RESISTORS


High ohm resistors are sometimes used instead of diodes. A resistor is more durable than a
diode and can suppress voltage spikes similar to a diode, but the resistor will allow current
to flow through it whenever the relay is on. Therefore resistance of the resistor must be
fairly high (about 600 ohms) in order to prevent too much current flow in the circuit. High
ohm resistors are not quite as efficient at suppressing a voltage spike as diodes.
CIRCUIT IDENTIFICATION
Relays are easy to test but often misunderstood. Using a 4 pin relay for our example, we
must first identify the pins. Some manufacturers place a diagram and pin ID on the outside
of the relay case to show which pins are part of the control circuit and which pins are part of
the load circuit.

CONTINUITY CHECK FOR PIN ID


If the relay is not labeled, use an ohmmeter and check to see which pins are connected to
each other. You should typically find an ohm value of approximately 50 to 120 ohms
between two of the pins. This is the control circuit. If the coil is less that 50 ohms it could be
suspect. Refer to manual to verify reading. The remaining two pins should read OL (infinite)
if it's a normally open relay, or 0 ohms (continuity) if it's a normally closed relay. If the
readings are correct, proceed to the next test. Note: If none of the relay pins showed a coil
value and all pins show OL or 0 ohms, the control coil is damaged and should be replaced.
PRACTICAL TESTING
Once the pins have been identified, energize the control circuit by supplying B+ to pin 1 and
a ground to pin 3. A faint "click" will be heard; although this "click" means the switch has
moved (closed), it does not mean the relay is good. The load circuit switch contacts could
still be faulty (high resistance), and further testing is required. A common mistake
technicians make is they hear a "click" and assume the relay is good. Take the extra step
and verify operation.

CAUTION
Testing relays with built in clamping diodes require a special procedure. These relays are
polarity sensitive; placing B+ to the wrong pin (backwards) while performing a practical test
will forward bias the diode and damage the diode, thus destroying the protective quality of
the diode.
OPERATIONAL CHECK WITH TESTLIGHT
Now start the second part of the test. Energize the relay (control side) by supplying B+ to
pin 1 and a ground to pin 3. A click should be heard. With the relay still energized, supply
B+ pin 2 of the load circuit. The test light will be on. De-energize (remove B+) the control
circuit at pin 1; the test light at pin 4 should go off. A test light is preferred because a test
light will draw current through the switch.

CAUTION
Testing relays with built in clamping diodes require a special procedure. These relays are
polarity sensitive; placing B+ to the wrong pin (backwards) while performing a practical test
will forward bias the diode and damage the diode, thus destroying the protective quality of
the diode.

OPERATIONAL CHECK WITH VOLTMETER


A voltmeter can be substituted in place of a test light; however be aware if the contacts are
partially burned, the voltmeter will show voltage indicating good contact even when bad.
Remember high impedance digital voltmeters draw almost no current. Energize the relay
(control side) by supplying B+ to pin 1 and a ground to pin 3. A click should be heard. With
the relay still energized supply B+ to pin 2 of the load circuit. Connect the RED lead to pin 4
and the BLACK lead to ground. The voltmeter will indicate source voltage (12V). De-
energize (remove B+) the control circuit at pin 1; the voltmeter should now read "zero". Re-
energize the relay and the voltmeter should return to 12 volts.
CAUTION
Testing relays with built in clamping diodes require a special procedure. These relays are
polarity sensitive; placing B+ to the wrong pin (backwards) while performing a practical test
will forward bias the diode and damage the diode, thus destroying the protective quality of
the diode.

OPERATIONAL CHECK WITH AN OHMMETER


An ohmmeter can also be used to test the load circuit, but the same problem as the
voltmeter comes into play. Energize the relay (control side). Supply B+ to pin 1 and a
ground (neg.) to pin 3. A click should be heard. Place the leads on an ohmmeter to across
pin 2 and pin 4. Assuming it is a normally open relay the ohmmeter will indicate a complete
circuit (close to zero -0 ohms). De-energize the control circuit at pin 1(remove B+). The
ohmmeter should indicate OL (an open circuit - infinite). Re-energize the relay and the
ohmmeter should return to "zero" ohms. Note: some manufactures provide a maximum
ohm value when the switch contacts are closed, example 5 ohms max.
CAUTION
Testing relays with built in clamping diodes require a special procedure. These relays are
polarity sensitive; placing B+ to the wrong pin (backwards) while performing a practical test
will forward bias the diode and damage the diode, thus destroying the protective quality of
the diode.

OPERATIONAL CHECK FOR RELAY VOLTAGE SUPPRESSION DIODES


An ANALOG OHMMETER must be used. This test cannot be performed with a digital meter.
The analog meter sends out a higher voltage which is required to forward bias the diode.
Place the ohmmeter across the control circuit and record reading. Reverse the leads and
check the control circuit again. A functioning diode will be indicated by have two different
readings. A faulty diode will have the same reading in both directions.

Current from the ohmmeter flows through the control coil, in one direction. By reversing the
leads, you send current in the opposite direction through the control coil. One of the two
directions the diode will be forward biased(on), creating two paths for current thus lowering
resistance. With the leads in the other direction, the diode in will be reversed biased (off)
creating only one path, with higher resistance.

STUDY OF SYMBOLS,ACCESSORIES,TOOLS USED IN WIRING AND SAFETY


PRECAUTIONS
AIM
To
ACCESSORIES REQUIRED
TOOLS REQUIRED
ACCESSORIES
TOOLS
STUDY OF SYMBOLS
SAFETY PRECAUTIONS

The following practices may reduce risk of injury or fire when working with electrical
equipment:

Avoid contact with energized electrical circuits.


Use guarding around exposed circuits and sources of live electricity.
Disconnect the power source before servicing or repairing electrical equipment.
When it is necessary to handle equipment that is plugged in, be sure hands are dry and,
when possible, wear nonconductive gloves and shoes with insulated soles.
If it is safe to do so, work with only one hand, keeping the other hand at your side or in
your pocket, away from all conductive material. This precaution reduces the likelihood of
accidents that result in current passing through the chest cavity.
Minimize the use of electrical equipment in cold rooms or other areas where
condensation is likely. If equipment must be used in such areas, mount the equipment on
a wall or vertical panel.
If water or a chemical is spilled onto equipment, shut off power at the main switch or
circuit breaker and unplug the equipment.
If an individual comes in contact with a live electrical conductor, do not touch the
equipment, cord or person. Disconnect the power source from the circuit breaker or pull
out the plug using a leather belt.

WIRING DIAGRAM
RESULT

RESIDENTIAL HOUSE WIRING USING SWITCHES,FUSE,INDICATOR,LAMP AND


ENERGY METER
CIRCUIT DIAGRAM
LAYOUT DIAGRAM
AIM
To construct house wiring using switches,fuse,indicator,lamp and energy meter.

APPARATUS REQUIRED

S.NO APPARATUS RANGE QUANTITY

1 Energy meter (240V,(5-10)A, 1

2 Fuse 5A 1

3 Indicator - 1

4 Switches 5A 3

5 Lamp 60W 3

THEORY
PRECAUTIONS
PROCEDURE:
(i) Connections are made as per the circuit diagram
(ii) Switch ON the AC supply
(iii) The power supply supply is given to the circuit and the bulb was being made to
glow by operating the switch.
RESULT

ONE LAMP CONTROLLED BY ONE WAY SWITCH


CIRCUIT DIAGRAM
LAYOUT DIAGRAM
AIM

APPARATUS REQUIRED
THEORY
PRECAUTIONS
PROCEDURE
RESULT
FLUORESCENT LAMP WIRING
CIRCUIT DIAGRAM
LAYOUT DIAGRAM
AIM
APPARATUS REQUIRED
THEORY
PRECAUTIONS
PROCEDURE
RESULT
STAIRCASE WIRING
CIRCUIT DIAGRAM
LAYOUT DIAGRAM
AIM
APPARATUS REQUIRED
THEORY
PRECAUTIONS
PROCEDURE
RESULT
MEASUREMENT OF ELECTRICAL QUANTITIES- VOLTAGE,CURRENT,POWER
AND POWER FACTOR IN RLC CIRCUIT
CIRCUIT DIAGRAM
TABULATION
CALCULATION
AIM
APPARATUS REQUIRED
THEORY
PRECAUTIONS
PROCEDURE
FORMULA USED
RESULT
MEASUREMENT OF ENERGY USING SINGLE PHASE ENERGY METER
CIRCUIT DIAGRAM
TABULATION
CALCULATION
AIM
APPARATUS REQUIRED
THEORY
PRECAUTIONS
PROCEDURE
FORMULA USED
RESULT
MEASUREMENT OF RESISTANCE TO EARTH OF AN ELECTRICAL EQUIPMENT
BLOCK DIAGRAM
CIRCUIT DIAGRAM
TABULATION
CALCULATION
AIM
APPARATUS REQUIRED
THEORY
PRECAUTIONS
PROCEDURE
FORMULA USED
RESULT
CALIBRATION OF AMMETER
CIRCUIT DIAGRAM
TABULATION
CALCULATION
AIM
APPARATUS REQUIRED
THEORY
PRECAUTIONS
PROCEDURE
FORMULA USED
RESULT

CALIBRATION OF VOLTMETER
CIRCUIT DIAGRAM
TABULATION
CALCULATION
AIM
APPARATUS REQUIRED
THEORY
PRECAUTIONS
PROCEDURE
FORMULA USED
RESULT
Aim:
To study Full- Wave Rectifier.
To find its:

1. Percentage Regulation
2. Ripple Factor
3. Efficiency

Components:

Name Quantity

Diodes 1N4007(Si) 1

Resistor 1K 1

Capacitor 1000F 1
Equipment:

Name Range Quantity

CRO 0-20 MHz 1

Multimeter 1

Transformer 220V/9V, 50Hz 1

Connecting Wires
Theory:
The conversion of AC into DC is called Rectification. Electronic Devices can convert AC power
into DC power with high efficiency.
The full-wave rectifier consists of a center-tap transformer, which results in equal voltages above
and below the center-tap. During the positive half cycle, a positive voltage appears at the anode
of D1 while a negative voltage appears at the anode of D2. Due to this diode D1 is forward
biased it results in a current Id1 through the load R.
During the negative half cycle, a positive voltage appears at the anode of D2 and hence it is
forward biased. Resulting in a current Id2 through the load at the same instant a negative voltage
appears at the anode of D1 thus reverse biasing it and hence it doesnt conduct.
Ripple Factor:
Ripple factor is defined as the ratio of the effective value of AC components to the average DC
value. It is denoted by the symbol ' '.

Rectification Factor:
The ratio of output DC power to input Ac power is defined as efficiency.

, = 81%(if R >> Rf, then Rf can be neglected).


Percentage of Regulation:
It is a measure of the variation of AC output voltage as a function of DC output voltage.

Percentage of regulation = %
VNL = Voltage across load resistance, when minimum current flows through it.
VFL = Voltage across load resistance, when maximum current flows through.
For an ideal full wave rectifier, the percentage regulation is 0 percent. The percentage of
regulation is very small for a practical full wave rectifier.
Peak- Inverse Voltage (PIV):
It is the maximum voltage that has to be with stood by a diode when it is reverse biased
PIV = 2Vm
Advantages of Full wave Rectifier:

1. is reduced.
2. is improved.

Disadvantages of Full wave Rectifier:

1. Output voltage is half the secondary voltage.


2. Diodes with high PIV rating are used.

Manufacturing of the center-tapped transformer is quite expensive and so Full wave rectifier
with center-tapped transformer is costly.
Circuit Diagram:
Half Wave Rectifier (without filter):

Half Wave Rectifier (with filter):

Procedure:

1. Connect the circuit as shown in the circuit diagram.


2. Connect the primary side of the transformer to AC mains and the secondary side to
rectifier input.
3. Using a CRO, measure the AC input voltage of the rectifier, AC and DC voltage at the
output of the rectifier.
4. Observe the Waveforms at the secondary windings of transformer and across load
resistance.

Calculations:

1. Ripple Factor
2. Ripple factor with filter is

3. Percentage Regulation = %
Expected Waveforms:

Result:
Full Wave rectifier characteristics are studied.

Viva Questions:
1. What is filter ?
Ans: Electronic filters are electronic circuits which perform signal processing functions,
specifically to remove unwanted frequency components from the signal.
2. Give some rectifications technologies?
Ans: Synchronous rectifier, Vibrator, Motor-generator set , Electrolytic ,Mercury arc, and Argon
gas electron tube.
3. What is the efficiency of bridge rectifier?

Ans: %
4. PIV center tapped FWR?
Ans: 2Vm.
5. In filters capacitor is always connected in parallel, why?
Ans: Capacitor allows AC and blocks DC signal.in rectifier for converting AC to DC, capacitor
placed in parallel with output, where output is capacitor blocked voltage.If capacitance value
increases its capacity also increases which increases efficiency of rectifier.
Aim:
To study Half- Wave Rectifier.
To find its:

1. Percentage Regulation
2. Ripple Factor
3. Efficiency

Components:

Name Quantity

Diodes1N4007(Si) 1

Resistor 1K 1

Capacitor 100F 1
Equipment:

Name Range Quantity

CRO 0-20MHz 1
Name Range Quantity

Multimeter 1

Transformer 220V/9V, 50Hz 1

Connecting Wires
Theory:
The conversion of AC into DC is called Rectification. Electronic Devices can convert AC power
into DC power with high efficiency.
Consider the given circuit. Assume the diode to be ideal i.e., Vf = 0, Rr = Infinite, Rs = 0. During
the positive half cycle, the diode is forward biased and it conducts and hence a current flows
through the load resistor. During the negative half cycle, the diode is reverse biased and it is
equivalent to an open circuit, hence the current through the load resistance is zero. Thus the
diode conducts only for one half cycle and results in a half wave rectified output.
Theoretical calculations for Ripple Factor:
Without Filter:
Vrms= Vm/2
Vdc= Vm/

Ripple factor =
With Filter:

Ripple Factor
Where
f = 50Hz
R = 1K
Percentage of Regulation:
It is a measure of the variation of AC output voltage as a function of DC output voltage.

Percentage of regulation %
VNL = Voltage across load resistance, when minimum current flows through it.
VFL = Voltage across load resistance, when maximum current flows through.
For an ideal half-wave rectifier, the percentage regulation is 0 percent. For a practical half-wave
rectifier.

Converting Idc into its corresponding Im value and substituting in the percentage of regulation
formula we get.

Percentage of regulation %
Since Rf + R is small as compared to RL. The percentage of regulation is very small for half-
wave rectifier.
Circuit Diagram:
Half Wave Rectifier (without filter):

Half Wave Rectifier (with filter):


Procedure:

1. Connect the circuit as shown in the circuit diagram.


2. Connect the primary side of the transformer to AC mains and the secondary side to
rectifier input.
3. Using a CRO, measure the AC input voltage of the rectifier, AC and DC voltage at the
output of the rectifier.
4. Observe the Waveforms at the secondary windings of transformer and across load
resistance.

Calculations:

Ripple Factor

Ripple Factor with filter

Percentage Regulation %
Expected Waveforms:
Result:
Half Wave rectifier characteristics with and without filter are studied.

Viva Questions:
1. What is a rectifier?
Ans: A rectifier is an electrical device that converts alternating current (AC), which periodically
reverses direction, to direct current (DC), which flows in only one direction. The process is
known as rectification.
2. What is a ripple factor?
Ans: Ripple factor can be defined as the variation of the amplitude of DC (Direct current) due to
improper filtering of AC power supply. it can be measured by RF = vrms / vdc
3. What is efficiency?
Ans: Rectifier efficiency is the ratio of the DC output power to the AC input power.
4. What is PIV?
Ans: The peak inverse voltage is either the specified maximum voltage that a diode rectifier can
block, or, alternatively, the maximum that a rectifier needs to block in a given application.
5. What are the applications of rectifier?
Ans: The primary application of rectifiers is to derive DC power from an AC supply. Virtually
all electronic devices require DC, so rectifiers are used inside the power supplies of virtually all
electronic equipment. Rectifiers are also used for detection of amplitude modulated radio signals.
ectifiers are used to supply polarised voltage for welding.

Potrebbero piacerti anche